PT8.S1.Q20 - Saunders: Everyone at last week's neighborhood

ogoewannogoewann Core Member
edited November 2023 in Logical Reasoning 16 karma

Stuck between question A and B. Any tips?

Admin Note: Edited title. For LR questions, please use the format: "PT#.S#.Q# - brief description of the question."

Comments

  • luffyyyyluffyyyy Live Member
    692 karma

    hi there!

    so the rule for A is "the action that would result in the most housing for people who need it should be adopted. Exception = the building is believed to pose a threat to neighbourhood safety". Do we fall inside the exception or outside the exception? We are inside the exception because we are told that these houses pose a threat to the safety of our neighbourhood. When we are inside the exception, the rule goes away, so this rule doesn't apply to us anymore. This means it is not helpful to establish whether the majority was right or if we should adopt what opponents advocated.

    For B, let's break it down! One of the proposals would preclude the possibility of trying the other approach. The two proposals are 1) take down the houses / demolition and 2) don't take down the houses / rehabilitation. So what B is saying is that if we go with Proposal 1 (take the houses down) then we can't try Proposal 2 (rehabilitation). And if the first proposal doesn't go well (i.e. it proves unsatisfactory) then we should do the one that doesn't prevent the other possibility from happening. In other words, B is saying "go with the proposal that doesn't prevent us from trying the other proposal". So this would help us to determine that the proposal advocated by the opponents should have been adopted.

    I hope this helps!

Sign In or Register to comment.